Your-Doctor
Multiple Choice Questions (MCQ)



Free Palestine
Quiz Categories Click to expand

Category: Q&A Medicine--->Practice Examination
Page: 6

Question 26# Print Question

A 68-year-old man complains of cold blue hands. He first noticed a color change in the fall and reports that it has persisted through the winter. Further questioning reveals that he has been experiencing drenching night sweats the past few months and intermittent fevers. His wife notes that he has lost about 10 kg from his usual 80 kg weight at that time. She also notes that he has become more tired than usual and no longer wants to play tennis with their friends. Both his spleen and liver are palpable on examination. Laboratory results as well as his peripheral blood smear (Figure below) are shown below.

  • Hemoglobin   9.4 g/dL
  • Leukocyte count   36,100/mm3
  • Platelets   140,000/mm3
  • LDH   410 U/L (normal range 140–280 U/L)
  • Haptoglobin   27 mg/dL (normal range 45–165 mg/dL)

What is the most likely cause of his chief complaint?

A. Raynaud phenomenon
B. Cold agglutinin disease
C. Thromboangiitis obliterans
D. Connective tissue disorder
E. Hashimoto thyroiditis


Question 27# Print Question

A 15-year-old boy is brought in by his mother with a 3-day history of dark urine. The patient feels well but reports having a severe sore throat 3 weeks ago that resolved on its own. Physical examination reveals a blood pressure of 142/88 mmHg and periorbital edema.

Which of the following is the underlying mechanism for this patient’s disease?

A. Basement membrane thickening
B. Type III hypersensitivity reaction
C. Foot process effacement
D. Mesangial proliferation


Question 28# Print Question

A 63-year-old woman presents to the hospital with cough and shortness of breath. She has not been to the doctor in years and has no medical history. Her symptoms have been present for the past few years, but have been unbearable over the last week when she developed a worsening cough that was productive of yellow sputum. She cannot walk 50 feet without becoming exceedingly short of breath. She drinks alcohol moderately and has a 40 pack-year smoking history. On examination, her temperature is 38.1°C, blood pressure is 108/62 mmHg, heart rate is 102 beats per minute, respiratory rate is 28 breaths per minute, and oxygen saturation is 91%. Her examination is notable for mild jugular venous distention, distant heart and breath sounds, and scattered wheezes and rhonchi.

Which of the following is NOT recommended as therapy at this time?

A. Furosemide
B. Methylprednisolone
C. Albuterol
D. Levofloxacin
E. Ipratropium


Question 29# Print Question

A 55-year-old man presents to the Emergency Department complaining of chest pain radiating to his left shoulder. ECG reveals ST segment elevation in leads II, III, and aVF. Serum troponins are elevated. Vital signs show a temperature of 37°C, a blood pressure of 101/63 mmHg, a heart rate of 65 beats per minute, and a respiratory rate of 16 breaths per minute. Physical examination reveals diaphoresis, elevated jugular venous pressure (JVP), and a late systolic murmur. Lungs are clear to auscultation.

Which of the following can safely be used to treat the patient’s condition without risk of worsening his hemodynamic status?

A. IV fluids
B. Morphine
C. β-blockers
D. Furosemide
E. Aspirin


Question 30# Print Question

A 52-year-old man is brought to the Emergency Department for sudden onset of chest pain that developed 30 minutes ago. He was sitting at home watching TV when he developed the chest pain. He had upper respiratory symptoms 1 week ago that resolved with supportive treatment. He has a long history of hypertension and hyperlipidemia. His current medications include lisinopril and atorvastatin. He has a temperature of 37.3°C, a blood pressure of 155/88 mmHg, a heart rate of 104 beats per minute, and a respiratory rate of 16 breaths per minute. He is anxious on examination, and has an S4 on cardiac examination. His lungs are clear to auscultation bilaterally. An initial ECG is performed (Figure below), and troponins are drawn and found to be negative.

What is the most likely diagnosis?

A. STEMI
B. NSTEMI
C. Aortic dissection
D. Pericarditis




Category: Q&A Medicine--->Practice Examination
Page: 6 of 20